- Fri Jan 21, 2011 12:00 am
#22936
Complete Question Explanation
Assumption. The correct answer choice is (E)
This stimulus concludes that some cases of high blood pressure are not stress-induced, because illnesses caused by stress can only be treated by reducing it, and some cases of high BP can be effectively treated with medicine. The argument can be summarized in the following way:
Notice the inconsistency between the premises and the conclusion: just because some instances of high blood pressure are treatable with medicine, it does not necessarily follow that they are not stress-induced. What if the medicine actually worked by reducing stress? The conclusion only follows if we assume that the medication used to treat some cases of high BP does not work by reducing stress (M TRS). This is precisely what the author states in answer choice (E).
Answer choice (A): While this answer choice supports the conclusion by advancing the notion that there is no causal link between stress and high BP, it is not an assumption required by the argument. It is only necessary that some cases of high BP are unrelated to stress, not all of them. Beware of answer choices that are too strong in the context of Assumption questions.
Answer choice (B): This answer choice weakens the conclusion and is incorrect.
Answer choice (C): Even if reduced stress reduced a person's responsiveness to medicine, the conclusion of the argument will not change. Since the argument does not require it, answer choice (C) is incorrect.
Answer choice (D): This may seem to be an attractive answer at first. Let's try the Assumption Negation technique: what if all conditions that are treatable by medicines are also treatable through the reduction of stress? Wouldn't this be consistent with the idea that all cases of high BP are potentially caused by stress? Not necessarily.
Remember that the necessary condition in the author's first premise requires that any illness caused by stress is treatable only by the reduction of stress. Having two effective treatments at our disposal (medicine and stress reduction) still means that the illness in question is not stress-induced. Because the conclusion is not weakened by the logical opposite of answer choice (D), this answer choice is not a necessary assumption for it.
Answer choice (E): This is the correct answer choice. See discussion above.
Assumption. The correct answer choice is (E)
This stimulus concludes that some cases of high blood pressure are not stress-induced, because illnesses caused by stress can only be treated by reducing it, and some cases of high BP can be effectively treated with medicine. The argument can be summarized in the following way:
- Premise (1): SII (stress-induced illness) TRS (treatment must reduce stress)
Premise (2): HBP M (some high blood pressure is treatable with medication)
Conclusion: HBP SII (some high blood pressure is not stress-induced)
Notice the inconsistency between the premises and the conclusion: just because some instances of high blood pressure are treatable with medicine, it does not necessarily follow that they are not stress-induced. What if the medicine actually worked by reducing stress? The conclusion only follows if we assume that the medication used to treat some cases of high BP does not work by reducing stress (M TRS). This is precisely what the author states in answer choice (E).
Answer choice (A): While this answer choice supports the conclusion by advancing the notion that there is no causal link between stress and high BP, it is not an assumption required by the argument. It is only necessary that some cases of high BP are unrelated to stress, not all of them. Beware of answer choices that are too strong in the context of Assumption questions.
Answer choice (B): This answer choice weakens the conclusion and is incorrect.
Answer choice (C): Even if reduced stress reduced a person's responsiveness to medicine, the conclusion of the argument will not change. Since the argument does not require it, answer choice (C) is incorrect.
Answer choice (D): This may seem to be an attractive answer at first. Let's try the Assumption Negation technique: what if all conditions that are treatable by medicines are also treatable through the reduction of stress? Wouldn't this be consistent with the idea that all cases of high BP are potentially caused by stress? Not necessarily.
Remember that the necessary condition in the author's first premise requires that any illness caused by stress is treatable only by the reduction of stress. Having two effective treatments at our disposal (medicine and stress reduction) still means that the illness in question is not stress-induced. Because the conclusion is not weakened by the logical opposite of answer choice (D), this answer choice is not a necessary assumption for it.
Answer choice (E): This is the correct answer choice. See discussion above.